Mathcenter Forum  

Go Back   Mathcenter Forum > คณิตศาสตร์โอลิมปิก และอุดมศึกษา > คณิตศาสตร์อุดมศึกษา
สมัครสมาชิก คู่มือการใช้ รายชื่อสมาชิก ปฏิทิน ข้อความวันนี้

ตั้งหัวข้อใหม่ Reply
 
เครื่องมือของหัวข้อ ค้นหาในหัวข้อนี้
  #16  
Old 04 พฤศจิกายน 2006, 11:01
Timestopper_STG's Avatar
Timestopper_STG Timestopper_STG ไม่อยู่ในระบบ
ลมปราณคุ้มครองร่าง
 
วันที่สมัครสมาชิก: 22 มกราคม 2006
ข้อความ: 256
Timestopper_STG is on a distinguished road
Send a message via MSN to Timestopper_STG
Post

ผมหาข้อไหนมาซ้ำหมดเลยนะครับ ...ข้าน้อยขออภัยข้าน้อยยังอ่อนต่อโลกนัก
ส่วนวิธีทำข้อ1ด้วยวิธีทางลำดับถ้าภายใน2วันนี้ยังไม่มีคนทำผมจะเฉลยให้ดูกันครับ
__________________
$$\int_{0}^{\frac{\pi}{2}}\frac{a\cos x-b\sin x}{a\sin x+b\cos x}dx=\ln\left(\frac{a}{b}\right)$$
BUT
$$\int_{0}^{\frac{\pi}{2}}\frac{a\cos x+b\sin x}{a\sin x+b\cos x}dx=\frac{\pi ab}{a^{2}+b^{2}}+\frac{a^{2}-b^{2}}{a^{2}+b^{2}}\ln\left(\frac{a}{b}\right)$$
ตอบพร้อมอ้างอิงข้อความนี้
  #17  
Old 05 พฤศจิกายน 2006, 22:34
Mastermander's Avatar
Mastermander Mastermander ไม่อยู่ในระบบ
กระบี่ประสานใจ
 
วันที่สมัครสมาชิก: 14 ตุลาคม 2005
ข้อความ: 796
Mastermander is on a distinguished road
Post

7.
\[
\begin{array}{l}
\sum\limits_{k = 1}^n {\frac{1}{k}} \approx \ln n + \gamma \\
\gamma = 0.577215664901533... \\
\sum\limits_{n = 1}^{1000} {\frac{1}{n}} \approx \ln 1000 + \gamma = 7.48497 \\
\end{array}
\]

8.
$$\lim_{n\to\infty} (8^n+4^n+2^n)^{1/3}-2^n$$
__________________
โลกนี้มีคนอยู่ 10 ประเภท คือ คนที่เข้าใจเลขฐานสอง และคนที่ไม่เข้าใจ
ตอบพร้อมอ้างอิงข้อความนี้
  #18  
Old 05 พฤศจิกายน 2006, 22:54
M@gpie's Avatar
M@gpie M@gpie ไม่อยู่ในระบบ
ลมปราณไร้สภาพ
 
วันที่สมัครสมาชิก: 09 ตุลาคม 2003
ข้อความ: 1,227
M@gpie is on a distinguished road
Post

8. แบบมั่วมา ตอบ $\frac{1}{3}$ รึเปล่าครับ
พิจารณา \[ \lim_{n\rightarrow \infty} [ (8^n+4^n+2^n)^{\frac{1}{3}} - 2^n ] = \lim_{n\rightarrow \infty} 2^n \left[ \left(1+\frac{1}{2^n} +\frac{1}{2^{2n}}\right) ^{\frac{1}{3}} -1 \right] \]
ให้ $x-1=\frac{1}{2^n}$ จะได้ว่า $\frac{1}{2^{2n}} = x^2-2x+1$ จะเห็นว่า เมื่อ $n \rightarrow \infty$ จะได้ว่า $x \rightarrow 1$
จะได้ว่า \[ \lim_{n\rightarrow \infty} [ (8^n+4^n+2^n)^{\frac{1}{3}} - 2^n ] = \lim_{x \rightarrow 1} \frac{ \sqrt[3]{x^2-x+1} - 1 }{x-1 } = \frac{1}{3} \]
9. ให้ $a_0=1$ และ \[ a_{n+1} = \frac{1}{2} \left( a_n+\frac{2}{a_n}\right), n \geq 1 \]
จงแสดงว่าลำดับนี้ลู่เข้าและลู่เข้าสู่ค่าใด?

สงสัยจะเบลอไปนิด แก้ไขแล้วครับ แหะๆ
__________________
PaTa PatA pAtA Pon!

06 พฤศจิกายน 2006 14:16 : ข้อความนี้ถูกแก้ไขแล้ว 3 ครั้ง, ครั้งล่าสุดโดยคุณ M@gpie
ตอบพร้อมอ้างอิงข้อความนี้
  #19  
Old 06 พฤศจิกายน 2006, 00:08
passer-by passer-by ไม่อยู่ในระบบ
ผู้พิทักษ์กฎทั่วไป
 
วันที่สมัครสมาชิก: 11 เมษายน 2005
ข้อความ: 1,442
passer-by is on a distinguished road
Post

คุณ M@gpie เขียนโจทย์ผิดหรือเปล่าครับ
__________________
เกษียณตัวเอง ปลายมิถุนายน 2557 แต่จะกลับมาเป็นครั้งคราว
ตอบพร้อมอ้างอิงข้อความนี้
  #20  
Old 06 พฤศจิกายน 2006, 12:43
warut warut ไม่อยู่ในระบบ
กระบี่ไร้สภาพ
 
วันที่สมัครสมาชิก: 24 พฤศจิกายน 2001
ข้อความ: 1,627
warut is on a distinguished road
Post

อย่างที่คุณ passer-by บอก โจทย์ข้อ 9. ของคุณ M@gpie น่าจะผิดครับ

ให้ $\{a_n\}$ ลู่เข้าหา $a$ เราจะได้ว่า $a= \frac12 \left( a - \frac{2}{a} \right)$ นั่นคือ $a^2=-2$ ซึ่งไม่มีคำตอบเป็นจำนวนจริง ดังนั้น $\{a_n\}$ ลู่ออกครับ
ตอบพร้อมอ้างอิงข้อความนี้
  #21  
Old 08 พฤศจิกายน 2006, 07:13
warut warut ไม่อยู่ในระบบ
กระบี่ไร้สภาพ
 
วันที่สมัครสมาชิก: 24 พฤศจิกายน 2001
ข้อความ: 1,627
warut is on a distinguished road
Post

อ้างอิง:
ข้อความเดิมของคุณ M@gpie:
$$ \lim_{x \rightarrow 1} \frac{ \sqrt[3]{x^2-x+1} - 1 }{x-1 } = \frac{1}{3} $$
ตรงนี้คุณ M@gpie ใช้ L'Hospital's rule หรืออะไรครับ
ตอบพร้อมอ้างอิงข้อความนี้
  #22  
Old 08 พฤศจิกายน 2006, 20:10
M@gpie's Avatar
M@gpie M@gpie ไม่อยู่ในระบบ
ลมปราณไร้สภาพ
 
วันที่สมัครสมาชิก: 09 ตุลาคม 2003
ข้อความ: 1,227
M@gpie is on a distinguished road
Post

ก็ ถ้าให้ถูกต้องก็ควรใช้ factor $\frac{\sqrt[3]{(x^2-x+1)^2 }+\sqrt[3]{(x^2-x+1)}+1}{\sqrt[3]{(x^2-x+1)^2 }+\sqrt[3]{(x^2-x+1)}+1}$ ครับผม

แต่ถ้าแอบคิดในใจ ผมก็ใช้โลปิตาลครับ อิอิ
__________________
PaTa PatA pAtA Pon!
ตอบพร้อมอ้างอิงข้อความนี้
  #23  
Old 08 พฤศจิกายน 2006, 20:34
warut warut ไม่อยู่ในระบบ
กระบี่ไร้สภาพ
 
วันที่สมัครสมาชิก: 24 พฤศจิกายน 2001
ข้อความ: 1,627
warut is on a distinguished road
Post

อ๋อ เข้าใจแล้วครับ ถ้างั้นรวมๆแล้วก็คล้ายๆกับที่ผมทำ $$ \lim_{n\to\infty} (8^n+4^n+2^n)^{1/3} -2^n $$ $$= \lim_{n\to\infty} \frac{1+ \frac{1}{2^n} }{ \left( 1 + \frac{1}{2^n} + \frac{1}{4^n} \right) ^{2/3} + \left( 1 + \frac{1}{2^n} + \frac{1}{4^n} \right) ^{1/3} +1} $$ $$= \frac{1}{1+ 1+ 1} = \frac13 $$
ตอบพร้อมอ้างอิงข้อความนี้
  #24  
Old 13 พฤศจิกายน 2006, 17:48
Mastermander's Avatar
Mastermander Mastermander ไม่อยู่ในระบบ
กระบี่ประสานใจ
 
วันที่สมัครสมาชิก: 14 ตุลาคม 2005
ข้อความ: 796
Mastermander is on a distinguished road
Post

10.$$\sum_{n=1}^\infty \frac{n^2}{n^4+4}$$
__________________
โลกนี้มีคนอยู่ 10 ประเภท คือ คนที่เข้าใจเลขฐานสอง และคนที่ไม่เข้าใจ
ตอบพร้อมอ้างอิงข้อความนี้
  #25  
Old 13 พฤศจิกายน 2006, 20:07
passer-by passer-by ไม่อยู่ในระบบ
ผู้พิทักษ์กฎทั่วไป
 
วันที่สมัครสมาชิก: 11 เมษายน 2005
ข้อความ: 1,442
passer-by is on a distinguished road
Post

ข้อ 9

จะพิสูจน์ 2 อย่างคือ

(i) $ a_n ^2 \geq 2 \quad (n \geq 1) $

(ii) $ a_n-a_{n+1} \geq 0 \quad (n \geq 1)$

ซึ่งเพียงพอจะแสดง existence of limit

(i) $ a_{n} = \frac{1}{2} \left( a_{n-1}+\frac{2}{a_{n-1}}\right) \geq \frac{1}{2} \left( 2\sqrt{2}\right) =\sqrt{2} \quad (n \geq 1)$

(ii) ใช้ผลจาก (i)

ถ้า $ \lim_{n \rightarrow \infty} a_n = L $ ดังนั้น $ L= \frac{1}{2}\left(L+\frac{2}{L}\right ) \Rightarrow L =\sqrt{2} $

NOTE : เราสามารถ generalize เพื่อสร้าง recurrence relation ในการหา $ \sqrt{c} $ ได้เป็น

$ a_{n+1} = \frac{1}{2} \left( a_n+\frac{c}{a_n}\right) $

ต่อด้วย ข้อ 11.

ไม่อยากแปล เดี๋ยวจะงงกว่าเดิม

Let $ S_n $ be sum of products of the first n natural numbers, taken two at a time

Evaluate $$ \frac{2}{3!}+\frac{11}{4!}+\cdots \frac{S_{n-1}}{n!}+\cdots $$
__________________
เกษียณตัวเอง ปลายมิถุนายน 2557 แต่จะกลับมาเป็นครั้งคราว
ตอบพร้อมอ้างอิงข้อความนี้
  #26  
Old 13 พฤศจิกายน 2006, 23:16
Timestopper_STG's Avatar
Timestopper_STG Timestopper_STG ไม่อยู่ในระบบ
ลมปราณคุ้มครองร่าง
 
วันที่สมัครสมาชิก: 22 มกราคม 2006
ข้อความ: 256
Timestopper_STG is on a distinguished road
Send a message via MSN to Timestopper_STG
Icon17

11.$\because S_{n}=(1)(2)+(1)(3)+...+(n-1)n$
$=n(1+2+\cdots+(n-1))+(n-1)(1+2+\cdots+(n-2))+\cdots+2(1+2)$
$\displaystyle{=\frac{n^3-n^2}{2}+\frac{(n-1)^3-(n-1)^2}{2}+\cdots+\frac{2^3-2^2}{2}}$
$\displaystyle{=\frac{1}{24}((n-1)n(n+1)(3n+2))}$
ถ้าให้$a_n$คือพจน์ที่$n$ของอนุกรมดังกล่าวจะได้ว่า
$\displaystyle{a_n=\frac{S_{n+1}}{(n+2)!}=\frac{1}{24}\frac{3n+5}{(n-1)!}=\frac{1}{24}\left[( \frac{8}{(n-1)!}+\frac{3}{(n-2)!})\right]}$
$\displaystyle{\sum_{n=1}^{\infty}a_n=\sum_{n=1}^{\infty}\frac{8}{(n-1)!}+\sum_{n=2}^{\infty}\frac{3}{(n-2)!}}=\frac{11e}{24}$
__________________
$$\int_{0}^{\frac{\pi}{2}}\frac{a\cos x-b\sin x}{a\sin x+b\cos x}dx=\ln\left(\frac{a}{b}\right)$$
BUT
$$\int_{0}^{\frac{\pi}{2}}\frac{a\cos x+b\sin x}{a\sin x+b\cos x}dx=\frac{\pi ab}{a^{2}+b^{2}}+\frac{a^{2}-b^{2}}{a^{2}+b^{2}}\ln\left(\frac{a}{b}\right)$$
ตอบพร้อมอ้างอิงข้อความนี้
  #27  
Old 16 พฤศจิกายน 2006, 03:25
passer-by passer-by ไม่อยู่ในระบบ
ผู้พิทักษ์กฎทั่วไป
 
วันที่สมัครสมาชิก: 11 เมษายน 2005
ข้อความ: 1,442
passer-by is on a distinguished road
Smile

12.

ถ้า $ u_0 > 1 $ และ $ u_{n+1}=u_n^2-u_n+1 \,\, ,n \geq 0 $

(1) พิสูจน์ว่า $ \sum_{n=0}^{\infty} \frac{1}{u_n} $ ลู่เข้า

(2) ถ้า $ u_0 = 2 $ หาค่า $ \sum_{n=0}^{\infty} \frac{1}{u_n} $
__________________
เกษียณตัวเอง ปลายมิถุนายน 2557 แต่จะกลับมาเป็นครั้งคราว
ตอบพร้อมอ้างอิงข้อความนี้
  #28  
Old 18 พฤศจิกายน 2006, 19:39
warut warut ไม่อยู่ในระบบ
กระบี่ไร้สภาพ
 
วันที่สมัครสมาชิก: 24 พฤศจิกายน 2001
ข้อความ: 1,627
warut is on a distinguished road
Smile

โจทย์ข้อ 12. น่าสนใจมากครับ

(1) ผมขอเริ่มจากการพิสูจน์ว่า $$ u_0>1 \quad \Rightarrow \quad \lim_{n \to \infty} u_n = \infty $$ ให้ $v_n=u_n-1$ สำหรับทุกจำนวนเต็ม $n\ge0$

ดังนั้น $ v_{n+1} = v_n^2+v_n $ และ $v_0>0$ และเราจึงได้ว่า $v_n>0$ เสมอ

และจากที่ $ v_{n+1} - v_n = v_n^2 >0$ ดังนั้น $\{v_n\}$ จึงเป็น (strictly) increasing sequence

โดยใช้ induction เราจะพบว่า $v_n>nv_0^2$ (เพราะ $ v_{n+1} = v_n^2+v_n > v_0^2+nv_0^2 = (n+1)v_0^2 $ )

ดังนั้น $$ \lim_{n \to \infty} u_n = \lim_{n \to \infty} v_n+1 = \infty $$ ตามต้องการ และเนื่องจาก $$ \sum_{k=0}^n \frac{1}{u_k} = \sum_{k=0}^n \left( \frac{1}{u_k-1} - \frac{1}{u_{k+1}-1} \right) = \frac{1}{u_0-1} - \frac{1}{u_{n+1}-1} $$ ดังนั้น $$ \sum_{n=0}^\infty \frac{1}{u_n} = \frac{1}{u_0-1} $$ ซึ่งก็คือลู่เข้านั่นเองครับ

(2) แทนค่า $u_0=2$ ลงไปในสูตรข้างบน จะได้ผลบวกคือ 1 ครับ

ถ้ามีวิธีพิสูจน์แบบอื่นๆ ช่วยบอกกันด้วยนะครับ

19 พฤศจิกายน 2006 01:36 : ข้อความนี้ถูกแก้ไขแล้ว 1 ครั้ง, ครั้งล่าสุดโดยคุณ warut
ตอบพร้อมอ้างอิงข้อความนี้
  #29  
Old 18 พฤศจิกายน 2006, 21:30
passer-by passer-by ไม่อยู่ในระบบ
ผู้พิทักษ์กฎทั่วไป
 
วันที่สมัครสมาชิก: 11 เมษายน 2005
ข้อความ: 1,442
passer-by is on a distinguished road
Thumbs up

คุณ Warut แถมสูตร summation ให้เสร็จสรรพเลยนะครับ

สำหรับข้อ 12 ผมเอามาจาก การบ้านใน Review (Chapter 0) ของวิชา Functional Analysis ครับ

Alternative Solution:

(1)
Let $ u_0 = 1+ \epsilon \quad \exists \epsilon > 0 $

It's easy to show that $ u_n \geq 1+ \epsilon(1+ \epsilon)^n $ by induction

Since $\frac{1}{u_n} \leq \frac{1}{\epsilon(1+ \epsilon)^n} $ , it follows immediately from Comparison test and geometric series that $ \sum \frac{1}{u_n} $ converges

(2)

It's easy to show that $ u_n $ is positive increasing sequence.

By trying to compute partial sum of first 3-4 terms, we can guess that $$ \sum_{n=0}^N \frac{1}{u_n} = 1- \frac{1}{\prod_{i=0}^N u_i} $$ The above equation can be proved easily by induction on $ N $

By letting $ N \rightarrow \infty $ , it follows that $ \sum_{n=0}^{\infty} \frac{1}{u_n}=1 $

NOTE :
ถ้าสังเกตดีๆ series ที่ให้มาในข้อ 12.2 ก็คือ ความพยายามที่จะ represent 1 ในเทอมของ ผลรวม(อนันต์) ส่วนกลับ ของจำนวนนับ (infinite sum of reciprocal of natural number)
โดยเริ่มต้นจาก $ 1= \frac{1}{2}+\frac{1}{2} $
จากนั้นก็ extend ผลบวกออกไป โดยเลือกตัวที่ส่วนใกล้เคียง ส่วนของตัวหลังมากที่สุด ซึ่งก็คือ $\frac{1}{3} $ ดังนั้น $ 1= \frac{1}{2}+\frac{1}{3}+\frac{1}{6} $

แล้วก็ extend ผลบวกออกไปเรื่อยๆ โดยมองที่ $ \frac{1}{6} $ แล้วเลือกตัวที่ ส่วนใกล้เคียง 6 มากที่สุด ซึ่งก็คือ $ \frac{1}{7} $ ดังนั้น $ 1= \frac{1}{2}+\frac{1}{3}+\frac{1}{7}+\frac{1}{42} $ จากนั้นก็ repeat ขั้นตอนแบบเดิมไปเรื่อยๆครับ

จากการพิสูจน์ข้างต้นยืนยันได้ว่า infinite series ที่เราสร้างนี้ ลู่เข้าครับ
__________________
เกษียณตัวเอง ปลายมิถุนายน 2557 แต่จะกลับมาเป็นครั้งคราว

19 พฤศจิกายน 2006 19:55 : ข้อความนี้ถูกแก้ไขแล้ว 2 ครั้ง, ครั้งล่าสุดโดยคุณ passer-by
ตอบพร้อมอ้างอิงข้อความนี้
  #30  
Old 18 พฤศจิกายน 2006, 21:46
M@gpie's Avatar
M@gpie M@gpie ไม่อยู่ในระบบ
ลมปราณไร้สภาพ
 
วันที่สมัครสมาชิก: 09 ตุลาคม 2003
ข้อความ: 1,227
M@gpie is on a distinguished road
Post

ดีครับ เป็นความรู้ให้กับผมอย่างยิ่ง แหะๆ
__________________
PaTa PatA pAtA Pon!
ตอบพร้อมอ้างอิงข้อความนี้
ตั้งหัวข้อใหม่ Reply


หัวข้อคล้ายคลึงกัน
หัวข้อ ผู้ตั้งหัวข้อ ห้อง คำตอบ ข้อความล่าสุด
Alternating series (and Abel's theorem) Punk Calculus and Analysis 3 17 กรกฎาคม 2012 21:05
Marathon Mastermander ฟรีสไตล์ 6 02 มีนาคม 2011 23:19
On-Line Encyclopedia of Integer Sequences warut งานหรือข่าวคราวคณิตศาสตร์ทั่วไป 0 28 เมษายน 2007 00:28
ปัญหาชิงรางวัลข้อที่ 22: Infinite Series warut คณิตศาสตร์อุดมศึกษา 4 02 พฤศจิกายน 2006 05:35
Series intarapaiboon คณิตศาสตร์อุดมศึกษา 3 02 ตุลาคม 2005 10:58


กฎการส่งข้อความ
คุณ ไม่สามารถ ตั้งหัวข้อใหม่ได้
คุณ ไม่สามารถ ตอบหัวข้อได้
คุณ ไม่สามารถ แนบไฟล์และเอกสารได้
คุณ ไม่สามารถ แก้ไขข้อความของคุณเองได้

vB code is On
Smilies are On
[IMG] code is On
HTML code is Off
ทางลัดสู่ห้อง


เวลาที่แสดงทั้งหมด เป็นเวลาที่ประเทศไทย (GMT +7) ขณะนี้เป็นเวลา 06:28


Powered by vBulletin® Copyright ©2000 - 2024, Jelsoft Enterprises Ltd.
Modified by Jetsada Karnpracha